Inicio  1  2  3  4  5  6  7  8  9  10  11  12  13  14  15  16  17  18  19  20  21  22  23  24  25  26  27  28 

Ejercicios

  1. Calcule

    log28,      log93,      log2$ {\frac{1}{2}}$,      log10(0.0001).                          

  2. Exprese de manera que no aparezca el símbolo log

    log22n,      log29 + 4 log2$\displaystyle {\frac{1}{\sqrt{3}}}$,      log3$\displaystyle \left(\vphantom{ ab}\right.$ab$\displaystyle \left.\vphantom{ ab}\right)$ - log3$\displaystyle \left(\vphantom{ 3a}\right.$3a$\displaystyle \left.\vphantom{ 3a}\right)$ - log3$\displaystyle \left(\vphantom{ 9b}\right.$9b$\displaystyle \left.\vphantom{ 9b}\right)$.

  3. Halle los valores de x que satisfacen cada ecuación:

    log3x = 5,    log2$\displaystyle \left(\vphantom{ x+1}\right.$x + 1$\displaystyle \left.\vphantom{ x+1}\right)$ = - 1,    logx25 = 2.

  4. Calcule

    $\displaystyle \left(\vphantom{ \left( \sqrt{2}\right) ^{\sqrt{2}}}\right.$$\displaystyle \left(\vphantom{ \sqrt{2}}\right.$$\displaystyle \sqrt{2}$$\displaystyle \left.\vphantom{ \sqrt{2}}\right)^{\sqrt{2}}_{}$$\displaystyle \left.\vphantom{ \left( \sqrt{2}\right) ^{\sqrt{2}}}\right)^{\sqrt{2}}_{}$.

  5. Recordando que $ \sqrt{2}$ = 1. 414 213 562 3..., ordene en orden creciente los siguientes números

    21.42,  21.41,  2$\scriptstyle \sqrt{2}$,  21.412,  21.4139.

  6. Resuelva las siguientes ecuaciones exponenciales:

    $\displaystyle \begin{array}[c]{ccccc}%%
\left( \sqrt{2}\right) ^{x^{2}}=2 & 3^{...
...n2}=2^{\ln x} & \frac
{3^{x}-2^{x}}{2^{x}-1}=-1 & x^{2}=2^{\ln x} &
\end{array}$

  7. Resuelva las siguientes ecuaciones:

    1. ln(x2 + 1) = - x2,

    2. ex2 = 1 - x2,

    3. ex + e-x = 2,

    4. ex - e-x = 4.

  8. Demuestre usando el lema 4.1.1, que $ \left(\vphantom{ ab}\right.$ab$ \left.\vphantom{ ab}\right)^{x}_{}$ = axbx, para a, b > 0 y x $ \in$ IR.

  9. Demuestre que lim$ \left(\vphantom{ 1+\frac{1}{n}}\right.$1 + $ {\frac{1}{n}}$$ \left.\vphantom{ 1+\frac{1}{n}}\right)^{n+1}_{}$ = lim$ \left(\vphantom{ 1+\frac{1}{n+1}}\right.$1 + $ {\frac{1}{n+1}}$$ \left.\vphantom{ 1+\frac{1}{n+1}}\right)^{n}_{}$ = e.

  10. Si an $ \rightarrow$ l y f es continua en l, demuestre que f$ \left(\vphantom{ a_{n}}\right.$an$ \left.\vphantom{ a_{n}}\right)$ $ \rightarrow$ f$ \left(\vphantom{ l}\right.$l$ \left.\vphantom{ l}\right)$.

  11. Si $ \lim\limits_{x\rightarrow\infty}^{}$f (x) = l, y F es una función continua en l, demuestre que $ \lim\limits_{x\rightarrow\infty}^{}$(Fof )(x) = F(l ).

  12. Demuestre que lim$ \left(\vphantom{ 1+\frac{1}{n}}\right.$1 + $ {\frac{1}{n}}$$ \left.\vphantom{ 1+\frac{1}{n}}\right)^{a}_{}$ = 1, para a > 0. Sug. Considere ln$ \left(\vphantom{ 1+\frac{1}{n}}\right.$1 + $ {\frac{1}{n}}$$ \left.\vphantom{ 1+\frac{1}{n}}\right)^{a}_{}$.

  13. Demuestre que para a $ \geq$ 0 se tiene $ \lim\limits_{n\rightarrow\infty}^{}$$ \left(\vphantom{ 1+\frac{a}{n}}\right.$1 + $ {\frac{a}{n}}$$ \left.\vphantom{ 1+\frac{a}{n}}\right)^{n}_{}$ = ea. Sug. Para k = $ \left[\vphantom{ \frac{n}%%
{a}}\right.$$ {\frac{n}{a}}$$ \left.\vphantom{ \frac{n}%%
{a}}\right]$ justifique las siguientes desigualdades:

    $\displaystyle \left(\vphantom{ 1+\frac{1}{k+1}}\right.$1 + $\displaystyle {\frac{1}{k+1}}$$\displaystyle \left.\vphantom{ 1+\frac{1}{k+1}}\right)^{ka}_{}$ $\displaystyle \leq$ $\displaystyle \left(\vphantom{ 1+\frac{1}{k+1}}\right.$1 + $\displaystyle {\frac{1}{k+1}}$$\displaystyle \left.\vphantom{ 1+\frac{1}{k+1}}\right)^{n}_{}$ $\displaystyle \leq$ $\displaystyle \left(\vphantom{ 1+\frac{a}{n}}\right.$1 + $\displaystyle {\frac{a}{n}}$$\displaystyle \left.\vphantom{ 1+\frac{a}{n}}\right)^{n}_{}$ $\displaystyle \leq$ $\displaystyle \left(\vphantom{ 1+\frac{1}{k}}\right.$1 + $\displaystyle {\frac{1}{k}}$$\displaystyle \left.\vphantom{ 1+\frac{1}{k}}\right)^{n}_{}$ $\displaystyle \leq$ $\displaystyle \left(\vphantom{ 1+\frac{1}{k}}\right.$1 + $\displaystyle {\frac{1}{k}}$$\displaystyle \left.\vphantom{ 1+\frac{1}{k}}\right)^{a(k+1)}_{}$.

    Luego use los ejercicios 9 y 11, la continuidad de la función f (x) = xa.

  14. Use el ejercicio anterior, y la desigualdad de Bernoulli, para mostrar que ea $ \geq$ 1 + a,$ \forall$a $ \in$ IR.

  15. Demuestre que $ \lim\limits_{x\rightarrow\infty}^{}$$ \left(\vphantom{ 1+\frac{a}%%
{x}}\right.$1 + $ {\frac{a}{x}}$$ \left.\vphantom{ 1+\frac{a}%%
{x}}\right)^{x}_{}$ = ea. Sug. Escriba $ \left(\vphantom{ 1+\frac{a}{x}}\right.$1 + $ {\frac{a}{x}}$$ \left.\vphantom{ 1+\frac{a}{x}}\right)^{x}_{}$ = $ \left[\vphantom{ \left( 1+\frac{1}{t}\right) ^{t}}\right.$$ \left(\vphantom{ 1+\frac{1}{t}}\right.$1 + $ {\frac{1}{t}}$$ \left.\vphantom{ 1+\frac{1}{t}}\right)^{t}_{}$$ \left.\vphantom{ \left( 1+\frac{1}{t}\right) ^{t}}\right]^{a}_{}$, donde t = $ {\frac{a}{x}}$, y use el ejercicio 11 y la continuidad de la función x $ \mapsto$ xa.

  16. Demuestre que el resultado del ejercicio anterior sigue siendo válido para a < 0. Sug. Si a = - b, con b > 0, justifique lo siguiente:

    $\displaystyle \left(\vphantom{ 1+\frac{a}{x}}\right.$1 + $\displaystyle {\frac{a}{x}}$$\displaystyle \left.\vphantom{ 1+\frac{a}{x}}\right)^{x}_{}$ = $\displaystyle \left[\vphantom{ \left( 1+\frac{b}{x-b}\right)
^{x}}\right.$$\displaystyle \left(\vphantom{ 1+\frac{b}{x-b}}\right.$1 + $\displaystyle {\frac{b}{x-b}}$$\displaystyle \left.\vphantom{ 1+\frac{b}{x-b}}\right)^{x}_{}$$\displaystyle \left.\vphantom{ \left( 1+\frac{b}{x-b}\right)
^{x}}\right]^{-1}_{}$ $\displaystyle \rightarrow$ $\displaystyle \left(\vphantom{ e^{b}}\right.$eb$\displaystyle \left.\vphantom{ e^{b}}\right)^{-1}_{}$ = ea.

  17. Use la regla de L 'Hôpital, para mostrar que

    $\displaystyle \lim_{x\rightarrow\infty}^{}$$\displaystyle \left(\vphantom{ 1+\frac{a}{x}}\right.$1 + $\displaystyle {\frac{a}{x}}$$\displaystyle \left.\vphantom{ 1+\frac{a}{x}}\right)^{x}_{}$ = ea, para todo a $\displaystyle \in$ IR.

  18. Considere la sucesión xn = $ \left(\vphantom{ 1+\frac{1}{n}}\right.$1 + $ {\frac{1}{n}}$$ \left.\vphantom{ 1+\frac{1}{n}}\right)^{n}_{}$. Sabemos que lim xn = e.

    (a)
    Muestre que

    xn = 2 + $\displaystyle \sum_{k=2}^{n}$$\displaystyle \left(\vphantom{ 1-\frac{1}{n}}\right.$1 - $\displaystyle {\frac{1}{n}}$$\displaystyle \left.\vphantom{ 1-\frac{1}{n}}\right)$$\displaystyle \left(\vphantom{ 1-\frac{2}%%
{n}}\right.$1 - $\displaystyle {\frac{2}{n}}$$\displaystyle \left.\vphantom{ 1-\frac{2}%%
{n}}\right)$ ... $\displaystyle \left(\vphantom{ 1-\frac{k-1}{n}}\right.$1 - $\displaystyle {\frac{k-1}{n}}$$\displaystyle \left.\vphantom{ 1-\frac{k-1}{n}}\right)$$\displaystyle {\frac{1}{k!}}$$\displaystyle \forall$n $\displaystyle \geq$ 3,

    usando la fórmula del binomio.

    (b)
    Use la parte (a) para concluir que

    xn $\displaystyle \geq$ 2 + $\displaystyle \sum_{k=2}^{m}$$\displaystyle \left(\vphantom{ 1-\frac{1}{n}}\right.$1 - $\displaystyle {\frac{1}{n}}$$\displaystyle \left.\vphantom{ 1-\frac{1}{n}}\right)$$\displaystyle \left(\vphantom{ 1-\frac{2}%%
{n}}\right.$1 - $\displaystyle {\frac{2}{n}}$$\displaystyle \left.\vphantom{ 1-\frac{2}%%
{n}}\right)$ ... $\displaystyle \left(\vphantom{ 1-\frac{k-1}{n}}\right.$1 - $\displaystyle {\frac{k-1}{n}}$$\displaystyle \left.\vphantom{ 1-\frac{k-1}{n}}\right)$$\displaystyle {\frac{1}{k!}}$$\displaystyle \forall$n $\displaystyle \geq$ m.

    Luego tome el límite cuando n $ \rightarrow$ $ \infty$ para obtener que

    e $\displaystyle \geq$ $\displaystyle \sum\limits_{k=0}^{m}$$\displaystyle {\frac{1}{k!}}$$\displaystyle \forall$m $\displaystyle \in$ IN.

    (c)
    Concluya del teorema de Weierstrass que la sucesión $ \left(\vphantom{
S_{m}}\right.$Sm$ \left.\vphantom{
S_{m}}\right)$ definida por Sm = $\displaystyle \sum\limits_{k=0}^{m}$$ {\frac{1}{k!}}$ es convergente, y que

    $\displaystyle \lim_{n\rightarrow\infty}^{}$Sm $\displaystyle \leq$ e.

    (d)
    Use la parte (a) para mostrar que

    xn $\displaystyle \leq$ $\displaystyle \sum\limits_{k=0}^{n}$$\displaystyle {\frac{1}{k!}}$,

    y combine con la parte (c) para concluir que

    e = $\displaystyle \lim_{n\rightarrow\infty}^{}$$\displaystyle \sum\limits_{k=0}^{n}$$\displaystyle {\frac{1}{k!}}$ = : $\displaystyle \sum\limits_{k=0}^{\infty}$$\displaystyle {\frac{1}{k!}}$.

  19. Repita los pasos del ejercicio anterior, con xn = $ \left(\vphantom{ 1+\frac{a}{n}}\right.$1 + $ {\frac{a}{n}}$$ \left.\vphantom{ 1+\frac{a}{n}}\right)^{n}_{}$, para concluir que:

    ea = $\displaystyle \lim\limits_{m\rightarrow\infty}^{}$$\displaystyle \sum\limits_{k=0}^{m}$$\displaystyle {\frac{a^{k}}{k!}}$ = : $\displaystyle \sum\limits_{k=0}^{\infty}$$\displaystyle {\frac{a^{k}}{k!}}$, para a > 0.

    Sug. Use la fórmula del binomio para justificar lo siguiente:

     ea = lim$\displaystyle \left(\vphantom{ 1+\frac{a}{n}}\right.$1 + $\displaystyle {\frac{a}{n}}$$\displaystyle \left.\vphantom{ 1+\frac{a}{n}}\right)^{n}_{}$ $\displaystyle \geq$ lim$\displaystyle \sum\limits_{k=0}^{m}$$\displaystyle \choose{n}{k}$$\displaystyle {\frac{a^{k}}{n^{k}}}$ = $\displaystyle \sum\limits_{k=0}^{m}$$\displaystyle {\frac{a^{k}}{k!}}$.

    Luego considere la sucesión (Sn) definida por Sn = $ \sum\limits_{k=0}^{n}$$ {\frac{a^{k}}{k!}}$, con a > 0 fijo. Muestre que (Sn) es creciente y acotada superiormente, y concluya que (Sn) converge a cierto límite S $ \leq$ ea. Usando la fórmula del binomio otra vez se tiene

    xn $\displaystyle \leq$ $\displaystyle \sum\limits_{k=0}^{n}$$\displaystyle {\frac{a^{k}}{k!}}$ $\displaystyle \rightarrow$ ea,

    y concluya que S = ea.

  20. Resuelva las siguientes ecuaciones. Use una gráfica cuando lo amerite, para dar una idea inicial

    1. ln x = ln$ {\frac{1}{x+1}}$,

    2. ln x = x - 1,

    3. xex = 1,

    4. (ex)ex = 1,

    5. e2x - 3ex + 2 = 0,

    6. $ {\frac{e^{2x}-1}{5e^{x}-1}}$ = 1.

  21. Si f : IR $ \rightarrow$ R es continua, y f (x) = ax para todo x $ \in$ IQ, muestre que f (x) = ax, para todo x $ \in$ IR.

  22. Sea f : IR $ \rightarrow$ R monótona y tal que f (x) = ax, $ \forall$x $ \in$ IQ, muestre que f (x) = ax, $ \forall$x $ \in$ IR.

  23. Sea f : IR $ \rightarrow$ R, tal que f (x + y) = f (x)f (y), $ \forall$x, y $ \in$ IR, y que f (1) = a $ \neq$ 0.

    1. Muestre que f (x) $ \neq$ 0, $ \forall$x $ \in$ IR y que f (0) = 1.

    2. Muestre por inducción que f (nx) = f (x)n, para n $ \in$ IN y x $ \in$ IR.

    3. Concluya que f (n) = an, $ \forall$n $ \in$ IN.

    4. Muestre que f (- x) = $ {\frac{1}{f(x)}}$, $ \forall$x $ \in$ IR .

    5. Use (b) y (c) para concluir que f (r) = ar , $ \forall$r $ \in$ IQ.

    6. Si f es monótona o continua, entonces f (x) = ax, $ \forall$x $ \in$ IR (ver los ejercicios 21 y 22).

  24. Sean f, g : IR $ \rightarrow$ R continuas, y tales que f (r) = g(r), $ \forall$r $ \in$ IQ. Muestre que f = g (esto es, f (x) = g(x), $ \forall$x $ \in$ IR).

  25. Muestre que para b > 1, b $ \in$ IN, y x $ \in$ [0, 1] , existe una sucesión $ \left(\vphantom{ a_{n}}\right.$an$ \left.\vphantom{ a_{n}}\right)_{n\in I\!\! N}^{}$, con an $ \in$ {0,...b - 1}, tal que $ {\frac{a_{1}}{b}}$ +...+ $ {\frac{a_{n}}{b^{n}}}$ $ \leq$ x < $ {\frac{a_{1}}{b}}$ +...+ $ {\frac{a_{n}}{b^{n}}}$ + $ {\frac{1}{b^{n}}}$, $ \forall$n $ \in$ IN . Cuando b = 2, (an) se llama la expansión binaria de x.

  26. Sea ID = $ \left\{\vphantom{ \frac{k}{2^{n}}:k\in
I\!\! Z,n\in I\!\! N}\right.$$ {\frac{k}{2^{n}}}$ : k $ \in$ IZ, n $ \in$ IN$ \left.\vphantom{ \frac{k}{2^{n}}:k\in
I\!\! Z,n\in I\!\! N}\right\}$. Use el ejercicio anterior para mostrar que para todo x $ \in$ IR , existe una sucesión (rn) en ID, creciente y tal que

    rn $\displaystyle \leq$ x < rn + $\displaystyle {\frac{1}{2^{n}}}$,$\displaystyle \forall$n $\displaystyle \in$ IN.

    Además, para sn = rn + $ {\frac{1}{2^{n}}}$ se tiene que $ \left(\vphantom{
s_{n}}\right.$sn$ \left.\vphantom{
s_{n}}\right)$ es decreciente.

  27. Repita los ejercicios 21, 22 y 24, con ID en vez de IQ.

  28. Para todo x $ \in$ IR, existen dos sucesiones $ \left(\vphantom{ \tau
_{n}}\right.$$ \tau_{n}^{}$$ \left.\vphantom{ \tau
_{n}}\right)$ y ($ \sigma_{n}^{}$) de irracionales, tales que $ \left(\vphantom{ \tau
_{n}}\right.$$ \tau_{n}^{}$$ \left.\vphantom{ \tau
_{n}}\right)$ es creciente, $ \left(\vphantom{ \sigma_{n}}\right.$$ \sigma_{n}^{}$$ \left.\vphantom{ \sigma_{n}}\right)$ es decreciente, y $ \tau_{n}^{}$ $ \leq$ x $ \leq$ $ \sigma_{n}^{}$, para cada n. Sug. Considere la expansión decimal (o binaria, como en el ejercicio 26) de $ {\frac{x}{\sqrt{2}}}$.

  29. Calcule

    1. $ \lim\limits_{n\rightarrow\infty}^{}$$ \left(\vphantom{ 1-\frac{a^{2}}{n^{2}}}\right.$1 - $ {\frac{a^{2}}{n^{2}}}$$ \left.\vphantom{ 1-\frac{a^{2}}{n^{2}}}\right)^{n}_{}$ (sug. ea . e-a = 1).

    2. $ \lim\limits_{n\rightarrow\infty}^{}$$ \left(\vphantom{ 1+\frac{a}{n}}\right.$1 + $ {\frac{a}{n}}$$ \left.\vphantom{ 1+\frac{a}{n}}\right)^{n^{2}}_{}$,

    3. $ \lim\limits_{n\rightarrow\infty}^{}$$ \left(\vphantom{ 1+\frac{a}{n^{2}}}\right.$1 + $ {\frac{a}{n^{2}}}$$ \left.\vphantom{ 1+\frac{a}{n^{2}}}\right)^{n}_{}$, etc.

  30. Hallar el dominio máximo para las siguientes funciones:

    1. f (x) = ln(x2 - 2x + 3)

    2. g(x) = ln(x - 1) + ln(x + 3)

    3. h(x) = $ {\frac{\ln(x-1)}{\ln(x+3)}}$

    4. ln(x - 1)2

    5. 2 ln(x - 1)

    6. $ \sqrt{\ln(x-1)^{2}}$

    7. F(x) = $ \sqrt{1-e^{x}}$ + $ \sqrt{x}$

    8. G(x) = ln$ \sqrt{x^{2}-1}$ + $ \sqrt{\ln(x^{2}-1)}$

    9. J(x) = $ {\frac{1}{e^{x}-2}}$$ \sqrt{1-e^{x}}$

    10. k(x) = $ {\frac{1}{e^{x}-2}}$$ \sqrt{e^{x}-1}$

    11. H(x) = ln( 1 - $ \sqrt{x^{2}-1}$)

    12. L(x) = $ \sqrt{1-\sqrt{\ln x-1}}$

    13. M(x) = ln$ \left(\vphantom{ \ln\left( \ln\left( \ln x\right) \right) }\right.$ln$ \left(\vphantom{ \ln\left( \ln x\right) }\right.$ln$ \left(\vphantom{ \ln x}\right.$ln x$ \left.\vphantom{ \ln x}\right)$$ \left.\vphantom{ \ln\left( \ln x\right) }\right)$$ \left.\vphantom{ \ln\left( \ln\left( \ln x\right) \right) }\right)$.

  31. Sea xn = an, con a > 1. Demuestre que xn $ \rightarrow$ $ \infty$. ¿Qué pasa si 0 < a < 1?

  32. Demuestre que $ \lim\limits_{x\rightarrow\infty}^{}$ax = $ \infty$, si a > 1.

  33. Calcular:

    1. $ \lim\limits_{x\rightarrow0}^{}$$ {\frac{e^{x}-1}{x}}$. Sug. El límite coincide con la derivada de f (x) = ex, en x = 0.

    2. $ \lim\limits_{x\rightarrow0}^{}$$ {\frac{e^{x}-x-1}{x}}$.Sug. Use la parte (a).

  34. Para 0 $ \leq$ x $ \leq$ 1, demuestre que

    1 + x + $\displaystyle {\frac{x^{2}}{2}}$ $\displaystyle \leq$ ex $\displaystyle \leq$ 1 + x + $\displaystyle {\frac{x^{2}}{2}}$ + ex3.

    Sug. Recuerde que ex = $ \lim\limits_{n\rightarrow\infty}^{}$$ \left(\vphantom{
1+x+\ldots+\frac{x^{n}}{n!}}\right.$1 + x +...+ $ {\frac{x^{n}}{n!}}$$ \left.\vphantom{
1+x+\ldots+\frac{x^{n}}{n!}}\right)$. Además, para n $ \geq$ 3 se tiene

    $\displaystyle {\frac{x^{3}}{3!}}$...+ $\displaystyle {\frac{x^{n}}{n!}}$ = x3$\displaystyle \left(\vphantom{ \frac{1}{3!}%%
+\ldots+\frac{x^{n-3}}{n!}}\right.$$\displaystyle {\frac{1}{3!}}$ +...+ $\displaystyle {\frac{x^{n-3}}{n!}}$$\displaystyle \left.\vphantom{ \frac{1}{3!}%%
+\ldots+\frac{x^{n-3}}{n!}}\right)$ < ex3.

  35. Use el ejercicio anterior para demostrar que $ \lim\limits_{x\rightarrow0}^{}$$ {\frac{e^{x}-x-1}{x^{2}}}$ = $ {\frac{1}{2}}$.

  36. Derive las siguientes funciones

    f (x) = ln$ \left(\vphantom{ x^{2}+1}\right.$x2 + 1$ \left.\vphantom{ x^{2}+1}\right)$,  g(x) = $ \left(\vphantom{ e^{x^{2}+e^{x}}}\right.$ex2 + ex$ \left.\vphantom{ e^{x^{2}+e^{x}}}\right)^{x}_{}$,  h(x) = xx,  
    j(x) = $ \left(\vphantom{ \ln x}\right.$ln x$ \left.\vphantom{ \ln x}\right)^{\ln x}_{}$,  l (x) = $ \left(\vphantom{ \cos x}\right.$cos x$ \left.\vphantom{ \cos x}\right)^{x\ln x}_{}$.

  37. Calcule las siguientes integrales indefinidas

    $\displaystyle \begin{array}[c]{llll}%%
{\displaystyle\int}
xe^{x^{2}}dx\; &
{\d...
...frac{e^{x}dx}{e^{x}+1}\; &
{\displaystyle\int}
\frac{dx}{e^{x}+1}%%
\end{array}$

  38. Las funciones hiperbólicas se definen por

    cosh x = $\displaystyle {\frac{e^{x}+e^{-x}}{2}}$,  sinh x = $\displaystyle {\frac{e^{x}-e^{-x}}{2}}$,

    y luego

    tanh x = $\displaystyle {\frac{\sinh x}{\cosh x}}$,  coth x = $\displaystyle {\frac{1}{\tanh x}}$.

    Calcule las derivadas de cada una de estas funciones.

  39. Recuerde que

    ex = $\displaystyle \left(\vphantom{ \lim\sum\limits_{k=0}^{n}\frac{x^{k}}{k!}}\right.$lim$\displaystyle \sum\limits_{k=0}^{n}$$\displaystyle {\frac{x^{k}}{k!}}$$\displaystyle \left.\vphantom{ \lim\sum\limits_{k=0}^{n}\frac{x^{k}}{k!}}\right)$ $\displaystyle \geq$ $\displaystyle \sum\limits_{k=0}^{m}$$\displaystyle {\frac{x^{k}}{k!}}$,$\displaystyle \forall$m $\displaystyle \in$ IN.

    Use esto para mostrar que

    $\displaystyle \lim\limits_{x\rightarrow\infty}^{}$$\displaystyle {\frac{e^{x}}{p(x)}}$ = + $\displaystyle \infty$, (4.4)

    para todo polinomio p(x).

  40. En este ejercicio demostramos (4.4) de otra manera.

    1. Muestre que si r > 1 se tiene $ \lim\limits_{x\rightarrow\infty}^{}$$ {\frac{r^{x}}{x}}$ = $ \infty$. Sug. Siendo r = 1 + $ \varepsilon$, use el ejercicio 14 para concluir que

      rn > $\displaystyle {\frac{n(n-1)}{2}}$$\displaystyle \varepsilon^{2}_{}$.

      Luego, si n = $ \left[\vphantom{ x}\right.$x$ \left.\vphantom{ x}\right]$ tenemos

      rx $\displaystyle \geq$ rn $\displaystyle \geq$ $\displaystyle {\frac{n(n-1)}{2}}$$\displaystyle \varepsilon^{2}_{}$ $\displaystyle \geq$ $\displaystyle {\frac{(x-1)(x-2)}{2}}$$\displaystyle \varepsilon^{2}_{}$.

    2. Si r > 1 y p $ \in$ IR, muestre que $ \lim\limits_{x\rightarrow\infty}^{}$$ {\frac{r^{x}}{x^{p}}}$ = $ \infty$. Sug. Si p $ \leq$ 0 se tiene $ {\frac{r^{x}%%
}{x^{p}}}$ $ \geq$ rx, para x $ \geq$ 1. Si p > 0 entonces

      $\displaystyle \lim\limits_{x\rightarrow\infty}^{}$$\displaystyle {\frac{r^{x}}{x^{p}}}$ = $\displaystyle \lim\limits_{x\rightarrow\infty}^{}$$\displaystyle \left(\vphantom{ \frac{r^{x/p}}{x}}\right.$$\displaystyle {\frac{r^{x/p}}{x}}$$\displaystyle \left.\vphantom{ \frac{r^{x/p}}{x}}\right)^{p}_{}$,

      y use el ejercicio anterior con r1/p en vez de r.

    3. Use el ejercicio anterior para mostrar que

      $\displaystyle \lim\limits_{x\rightarrow\infty}^{}$$\displaystyle {\frac{P(x)}{r^{x}}}$ = 0,

      para cualquier polinomio P(x), y cualquier r > 1 (Cualquier exponencial de base mayor que 1, crece más rápido que cualquier polinomio). Note que esto es lo mismo que

      $\displaystyle \lim\limits_{x\rightarrow\infty}^{}$$\displaystyle {\frac{P(x)}{e^{\alpha x}}}$ = 0, $\displaystyle \forall$$\displaystyle \alpha$ > 0.

  41. Use el ejercicio anterior para demostrar que

    $\displaystyle \lim\limits_{x\rightarrow\infty}^{}$$\displaystyle {\frac{\left( \ln x\right) ^{\beta}}{P(x)}}$ = 0,

    para cualquier polinomio P(x), y cualquier $ \beta$ $ \in$ IR (el logaritmo crece mucho más lento que cualquier potencia). Sug. Haga el cambio de variable t = ln x.

  42. En este ejercicio se demuestra que e es irracional.

    1. Sea Sn = $\displaystyle \sum\limits_{k=0}^{n}$$ {\frac{1}{k!}}$. Demuestre que para n > m se tiene

      Sm < Sn = Sm + $ {\frac{1}{(m+1)!}}$$ \left[\vphantom{ 1+\frac{1}{m+2}+\frac{1}{(m+2)(m+3)}+\cdots}\right.$1 + $ {\frac{1}{m+2}}$ + $ {\frac{1}{(m+2)(m+3)}}$ + ... $ \left.\vphantom{ 1+\frac{1}{m+2}+\frac{1}{(m+2)(m+3)}+\cdots}\right]$
        < Sm + $ {\frac{1}{(m+1)!}}$$ \left[\vphantom{ 1+\frac{1}{m+1}+\frac{1}{(m+1)^{2}}+\cdots}\right.$1 + $ {\frac{1}{m+1}}$ + $ {\frac{1}{(m+1)^{2}}}$ + ... $ \left.\vphantom{ 1+\frac{1}{m+1}+\frac{1}{(m+1)^{2}}+\cdots}\right]$.

    2. Concluya que Sm < Sn < Sm + $ {\frac{1}{m\,m!}}$, si n > m. Haga n $ \rightarrow$ $ \infty$ para obtener

      Sm $\displaystyle \leq$ e $\displaystyle \leq$ Sm + $\displaystyle {\frac{1}{m\,m!}}$$\displaystyle \forall$m $\displaystyle \in$ IN.

    3. Tome m = 10, y use una calculadora de bolsillo para deducir que e = 2.7182818....Con n = 20, y una calculadora más eficiente, se obtiene e = 2.7182818284590452353...

    4. Suponga que e = $ {\frac{p}{m}}$, con p, m $ \in$ IN. Use este m en la estimación de arriba, y multiplique por m! para obtener

      mSm $\displaystyle \leq$ p(m - 1)! $\displaystyle \leq$ mSm + $\displaystyle {\frac{1}{m}}$.

      Como mSm es un entero, esto es una contradicción. Por lo tanto e es irracional.

  43. Sea f : IR $ \rightarrow$ IR una función derivable, tal que f$\scriptstyle \prime$(x) = f ((x), para todo x $ \in$ IR. Demuestre que existe C $ \in$ IR tal que f (x) = Cex, $ \forall$x $ \in$ IR. Sug. Derive la función g(x) = f (x)ex.


  Inicio  1  2  3  4  5  6  7  8  9  10  11  12  13  14  15  16  17  18  19  20  21  22  23  24  25  26  27  28